0 Daumen
148 Aufrufe

Hallo Leute

mir fällt das Lösen dieser Aufgabe etwas schwer. Ich muss für die Zufallsvariablen beweisen \( X_{n}, X: \Omega \rightarrow \mathbb{R}, n \in \mathbb{N}, \) die Áquivalenz der folgenden Aussagen:
(1) \( X_{n} \) konvergiert fast sicher gegen \( X \)
(2) \( \forall \epsilon>0: \lim \limits_{n \rightarrow \infty} P\left(\sup _{k \geq n}\left|X_{k}-X\right|>\epsilon\right)=0 \)

Ich denke für den Beweis benötigen wir eine Hilfsaussage.
Proposition 8.1.4. Seien (Ω, F) ein Messraum und E eine Menge. Außerdem seien X :

Allerdings weiß ich nicht, wie man das weiter fortführt, hat jemand eine Idee, wie man das löst? Vielen Dank im Voraus!

Liebe Grüße

Avatar von

Ein anderes Problem?

Stell deine Frage

Willkommen bei der Mathelounge! Stell deine Frage einfach und kostenlos

x
Made by a lovely community